Locally path connected implies path connected

  • Thread starter Thread starter Mosis
  • Start date Start date
  • Tags Tags
    Path
Click For Summary
The discussion centers on proving that an open and connected subset E of a locally path connected metric space X is also path connected. The initial approach involves considering the set Y of all points in E that are path connected to a fixed point x. To demonstrate that E is path connected, one must show that if Y does not encompass all of E, then the complement E-Y would be open, leading to a contradiction of the connectedness of E. The key insight is using the transitive property of path connectivity among points in Y. Ultimately, the problem requires clear thinking rather than complex ingenuity to reach the conclusion.
Mosis
Messages
53
Reaction score
0

Homework Statement


We say the metric space X is locally path connected (lpc) if all balls are path connected sets.

Suppose X is lpc and that E is an open and connected subset of X. Prove that E is path connected.

Homework Equations


A set is open if every point is an interior point. A set is connected if it cannot be written as the union of two open, disjoint, non-empty sets. A set is path connected if given any two points x,y in the set, there exists a path between them.


The Attempt at a Solution


None really. My professor suggested that for a fixed x, I consider the set Y of all y that are path connected to x and show that this set is all of E by somehow showing that if it were not, I could disconnect the space using Y and Y complement.

I just don't see the direction that this problem could take. Is it long? Is it short but requires ingenuity? What should I try to do? Where am I trying to end up?

Help, it's hard!
 
Physics news on Phys.org
It's not hard and doesn't require any stunning ingenuity. You just have to think clearly. Can you show Y is open? The main ingredient you need is that if a,b and c are three points, and a is path connected to b and b is path connected to c then a is path connected to c. Now can you show E-Y (the set of all points in E that aren't in Y) is also open? That's your disconnection.
 
Question: A clock's minute hand has length 4 and its hour hand has length 3. What is the distance between the tips at the moment when it is increasing most rapidly?(Putnam Exam Question) Answer: Making assumption that both the hands moves at constant angular velocities, the answer is ## \sqrt{7} .## But don't you think this assumption is somewhat doubtful and wrong?

Similar threads

  • · Replies 4 ·
Replies
4
Views
3K
Replies
20
Views
4K
  • · Replies 12 ·
Replies
12
Views
3K
  • · Replies 1 ·
Replies
1
Views
2K
  • · Replies 3 ·
Replies
3
Views
3K
  • · Replies 2 ·
Replies
2
Views
2K
  • · Replies 9 ·
Replies
9
Views
1K
  • · Replies 7 ·
Replies
7
Views
4K
Replies
6
Views
3K
  • · Replies 1 ·
Replies
1
Views
2K